Diễn Đàn MathScopeDiễn Đàn MathScope
  Diễn Đàn MathScope
Ghi Danh Hỏi/Ðáp Thành Viên Social Groups Lịch Ðánh Dấu Ðã Ðọc

Go Back   Diễn Đàn MathScope > Sơ Cấp > Đại Số và Lượng Giác > Các Bài Toán Đã Được Giải

News & Announcements

Ngoài một số quy định đã được nêu trong phần Quy định của Ghi Danh , mọi người tranh thủ bỏ ra 5 phút để đọc thêm một số Quy định sau để khỏi bị treo nick ở MathScope nhé !

* Nội quy MathScope.Org

* Một số quy định chung !

* Quy định về việc viết bài trong diễn đàn MathScope

* Nếu bạn muốn gia nhập đội ngũ BQT thì vui lòng tham gia tại đây

* Những câu hỏi thường gặp

* Về việc viết bài trong Box Đại học và Sau đại học


Trả lời Gởi Ðề Tài Mới
 
Ðiều Chỉnh Xếp Bài
Old 25-12-2010, 02:12 PM   #1
jakelong
+Thành Viên+
 
jakelong's Avatar
 
Tham gia ngày: Dec 2010
Đến từ: Baclieu
Bài gởi: 6
Thanks: 14
Thanked 1 Time in 1 Post
Gửi tin nhắn qua Yahoo chát tới jakelong
Th Minilacinho Một bài bất đẳng thức

Các pác Cm giùm em bài này với cách sử dụng Bất Đẳng Thức AM-GM Cho 2 Số Nhé
Cho $x,y,z > 0 $ và $xyz=1 $ Cm:
$(1+\frac{x}{y})(1+\frac{y}{z})(1+\frac{z}{x})\geq (1+x)(1+y)(1+z) $
[RIGHT][I][B]Nguồn: MathScope.ORG[/B][/I][/RIGHT]
 
__________________
Những viên kim cương trong bất đẳng thức toán học : http://img826.imageshack.us/img826/1198/51390270.jpg

thay đổi nội dung bởi: jakelong, 25-12-2010 lúc 02:23 PM
jakelong is offline   Trả Lời Với Trích Dẫn
Old 25-12-2010, 04:07 PM   #2
Lan Phuog
+Thành Viên Danh Dự+
 
Lan Phuog's Avatar
 
Tham gia ngày: Mar 2010
Đến từ: Thái Bình
Bài gởi: 564
Thanks: 289
Thanked 326 Times in 182 Posts
Trích:
Nguyên văn bởi jakelong View Post
Các pác Cm giùm em bài này với cách sử dụng Bất Đẳng Thức AM-GM Cho 2 Số Nhé
Cho $x,y,z > 0 $ và $xyz=1 $ Cm:
$(1+\frac{x}{y})(1+\frac{y}{z})(1+\frac{z}{x})\geq (1+x)(1+y)(1+z) $
Nhân khai triển và rút gọn thì bdt tương đương với:
$\frac{x}{z}+\frac{z}{y}+\frac{y}{x}\ge xy+yz+zx $
Sử dụng AM-GM như sau:
$\frac{x}{z}+\frac{z}{y}+\frac{z}{y}\ge 3\sqrt[3]{\frac{zx}{y^2}}=3zx $
Làm mấy cái tương tự rồi cộng vào thu được đpcm!
[RIGHT][I][B]Nguồn: MathScope.ORG[/B][/I][/RIGHT]
 
Lan Phuog is offline   Trả Lời Với Trích Dẫn
The Following 2 Users Say Thank You to Lan Phuog For This Useful Post:
avip (25-12-2010), jakelong (25-12-2010)
Old 25-12-2010, 04:22 PM   #3
avip
+Thành Viên+
 
Tham gia ngày: Sep 2010
Bài gởi: 392
Thanks: 135
Thanked 247 Times in 159 Posts
Trích:
Nguyên văn bởi Lan Phuog View Post
Nhân khai triển và rút gọn thì bdt tương đương với:
$\frac{x}{z}+\frac{z}{y}+\frac{y}{x}\ge xy+yz+zx $
Sử dụng AM-GM như sau:
$\frac{x}{z}+\frac{z}{y}+\frac{z}{y}\ge 3\sqrt[3]{\frac{zx}{y^2}}=3zx $
Làm mấy cái tương tự rồi cộng vào thu được đpcm!
Theo em thì khai triển ra thì cần cm $\sum_{sym} \frac{x}{y} \ge xy + yz + zx + x + y + z $ chứ ạ.
[RIGHT][I][B]Nguồn: MathScope.ORG[/B][/I][/RIGHT]
 
avip is offline   Trả Lời Với Trích Dẫn
The Following User Says Thank You to avip For This Useful Post:
jakelong (25-12-2010)
Old 25-12-2010, 04:29 PM   #4
Lan Phuog
+Thành Viên Danh Dự+
 
Lan Phuog's Avatar
 
Tham gia ngày: Mar 2010
Đến từ: Thái Bình
Bài gởi: 564
Thanks: 289
Thanked 326 Times in 182 Posts
Trích:
Nguyên văn bởi avip View Post
Theo em thì khai triển ra thì cần cm $\sum_{sym} \frac{x}{y} \ge xy + yz + zx + x + y + z $ chứ ạ.
ừ,nhầm 1 chút,nhưng k sao
cm thêm $\sum\frac{x}{y}\ge \sum x $
có: $\frac{x}{y}+\frac{x}{y}+\frac{y}{z}\ge 3x. $
ok!!!
[RIGHT][I][B]Nguồn: MathScope.ORG[/B][/I][/RIGHT]
 
Lan Phuog is offline   Trả Lời Với Trích Dẫn
The Following 2 Users Say Thank You to Lan Phuog For This Useful Post:
avip (25-12-2010), jakelong (25-12-2010)
Old 25-12-2010, 07:53 PM   #5
toanlc_gift
+Thành Viên+
 
toanlc_gift's Avatar
 
Tham gia ngày: Jan 2009
Đến từ: FU
Bài gởi: 171
Thanks: 31
Thanked 142 Times in 80 Posts
Trích:
Nguyên văn bởi jakelong View Post
Các pác Cm giùm em bài này với cách sử dụng Bất Đẳng Thức AM-GM Cho 2 Số Nhé
Cho $x,y,z > 0 $ và $xyz=1 $ Cm:
$(1+\frac{x}{y})(1+\frac{y}{z})(1+\frac{z}{x})\geq (1+x)(1+y)(1+z) $
đặt $x;y;z $ bằng $a^3;b^3;c^3 $
bđt trở thành
$({a^3} + {b^3})({b^3} + {c^3})({c^3} + {a^3}) \ge ({a^2} + bc)({b^2} + ac)({c^2} + ab) $,dễ thấy $VP \ge 8 $
thoe bđt Holder:
$({a^3} + {b^3})({c^3} + {b^3})({1^3} + {1^3}) \ge {(ac + {b^2})^3} $
làm 2 bđt tương tự rồi nhân lại ta đc
$8{\left( {VT} \right)^2} \ge {\left( {VP} \right)^3} \ge 8{(VP)^2} $
suy ra đpcm
[RIGHT][I][B]Nguồn: MathScope.ORG[/B][/I][/RIGHT]
 
toanlc_gift is offline   Trả Lời Với Trích Dẫn
The Following 3 Users Say Thank You to toanlc_gift For This Useful Post:
avip (25-12-2010), jakelong (25-12-2010), nhox12764 (25-12-2010)
Old 25-12-2010, 09:05 PM   #6
toanlc_gift
+Thành Viên+
 
toanlc_gift's Avatar
 
Tham gia ngày: Jan 2009
Đến từ: FU
Bài gởi: 171
Thanks: 31
Thanked 142 Times in 80 Posts
Trích:
Nguyên văn bởi jakelong View Post
Các pác Cm giùm em bài này với cách sử dụng Bất Đẳng Thức AM-GM Cho 2 Số Nhé
Cho $x,y,z > 0 $ và $xyz=1 $ Cm:
$(1+\frac{x}{y})(1+\frac{y}{z})(1+\frac{z}{x})\geq (1+x)(1+y)(1+z) $
quên mất còn giả thiết Bất Đẳng Thức AM-GM Cho 2 Số Nhé nữa ^^!
$(x + y)(x + z) = ({x^2} + yz) + x(y + z) \ge 2\sqrt {x({x^2} + yz)(y + z)} $
$= 2\sqrt {({x^3} + 1)(y + z)} = 2\sqrt {(x + 1)(\frac{{{{(x + 1)}^2}}}{4} + \frac{{3{{(x - 1)}^2}}}{4})(y + z)} $
$\ge \sqrt {{{(x + 1)}^3}(y + z)} $
làm 2 bđt tương tự rồi nhân lại và rút gọn ta đc đpcm
[RIGHT][I][B]Nguồn: MathScope.ORG[/B][/I][/RIGHT]
 
toanlc_gift is offline   Trả Lời Với Trích Dẫn
The Following 2 Users Say Thank You to toanlc_gift For This Useful Post:
jakelong (25-12-2010), wikipedia1995 (25-12-2010)
Old 25-12-2010, 09:25 PM   #7
Unknowing
+Thành Viên+
 
Unknowing's Avatar
 
Tham gia ngày: Mar 2010
Đến từ: THPT Hùng Vương Bình Phước( ۩xứ bụi ۩)
Bài gởi: 303
Thanks: 425
Thanked 302 Times in 164 Posts
Gửi tin nhắn qua Yahoo chát tới Unknowing
Trích:
Nguyên văn bởi jakelong View Post
Các pác Cm giùm em bài này với cách sử dụng Bất Đẳng Thức AM-GM Cho 2 Số Nhé
Cho $x,y,z > 0 $ và $xyz=1 $ Cm:
$(1+\frac{x}{y})(1+\frac{y}{z})(1+\frac{z}{x})\geq (1+x)(1+y)(1+z) $
[Only registered and activated users can see links. ]
cũng có bạn trả lời ở đây rồi

[RIGHT][I][B]Nguồn: MathScope.ORG[/B][/I][/RIGHT]
 
__________________
$Le~Thien~Cuong $
Unknowing is offline   Trả Lời Với Trích Dẫn
The Following User Says Thank You to Unknowing For This Useful Post:
jakelong (25-12-2010)
Old 26-12-2010, 12:18 AM   #8
No Problem
+Thành Viên+
 
Tham gia ngày: Feb 2010
Bài gởi: 29
Thanks: 9
Thanked 31 Times in 16 Posts
Trích:
Nguyên văn bởi jakelong View Post
Các pác Cm giùm em bài này với cách sử dụng Bất Đẳng Thức AM-GM Cho 2 Số Nhé
Cho $x,y,z > 0 $ và $xyz=1 $ Cm:
$(1+\frac{x}{y})(1+\frac{y}{z})(1+\frac{z}{x})\geq (1+x)(1+y)(1+z) $
$(1+\frac{x}{y})(1+xy)\ge \ (1+x)^2 $
<=>$(1+\frac{x}{y}).(\frac{1+z}{z})\ge \ (1+x)^2 $
Nhân lại vs nhau => đpcm
[RIGHT][I][B]Nguồn: MathScope.ORG[/B][/I][/RIGHT]
 
No Problem is offline   Trả Lời Với Trích Dẫn
The Following User Says Thank You to No Problem For This Useful Post:
jakelong (26-12-2010)
Trả lời Gởi Ðề Tài Mới

Bookmarks

Ðiều Chỉnh
Xếp Bài

Quuyền Hạn Của Bạn
You may not post new threads
You may not post replies
You may not post attachments
You may not edit your posts

BB code is Mở
Smilies đang Mở
[IMG] đang Mở
HTML đang Tắt

Chuyển đến


Múi giờ GMT. Hiện tại là 11:39 AM.


Powered by: vBulletin Copyright ©2000-2024, Jelsoft Enterprises Ltd.
Inactive Reminders By mathscope.org
[page compression: 71.30 k/80.89 k (11.85%)]